Difference between revisions of "1999 AIME Problems/Problem 2"

(complete solution, wikify)
Line 1: Line 1:
 
== Problem ==
 
== Problem ==
Consider the parallelogram with vertices <math>\displaystyle (10,45),</math> <math>\displaystyle (10,114),</math> <math>\displaystyle (28,153),</math> and <math>\displaystyle (28,84).</math>  A line through the origin cuts this figure into two congruent polygons.  The slope of the line is <math>\displaystyle m/n,</math> where <math>\displaystyle m_{}</math> and <math>\displaystyle n_{}</math> are relatively prime positive integers.  Find <math>\displaystyle m+n.</math>
+
Consider the [[parallelogram]] with [[vertex|vertices]] <math>\displaystyle (10,45),</math> <math>\displaystyle (10,114),</math> <math>\displaystyle (28,153),</math> and <math>\displaystyle (28,84).</math>  A [[line]] through the [[origin]] cuts this figure into two [[congruent]] [[polygon]]s.  The [[slope]] of the line is <math>\displaystyle m/n,</math> where <math>\displaystyle m_{}</math> and <math>\displaystyle n_{}</math> are [[relatively prime]] [[positive]] [[integer]]s.  Find <math>\displaystyle m+n.</math>
  
 
== Solution ==
 
== Solution ==
Let the first point on the line x=10 be (10,45+x) where x is the height above (10,45).  Let the second point (on the line x=28) be (28, 153-x). Since for a line to pass through the origin, and for two points (x,y) and (w,z) then <math>\displaystyle\frac{y}{x}=\frac{z}{w}</math>. Then 10(153-x)=28(45+x) and
+
Let the first point on the line <math>x=10</math> be <math>(10,45+a)</math> where a is the height above <math>(10,45)</math>.  Let the second point on the line <math>x=28</math> be <math>(28, 153-a)</math>. For two given points, the line will pass the origin iff the coordinates are [[proportion]]al (such that <math>\frac{y_1}{x_1} = \frac{y_2}{x_2}</math>). Then, we can write that <math>\frac{45 + a}{10} = \frac{153 - a}{28}</math>. Solving for <math>a</math> yields that <math>\displaystyle 1530 - 10a = 1260 + 28a</math>, so <math>a=\frac{270}{38}=\frac{135}{19}</math>. The slope of the line (since it passes through the origin) is <math>\frac{45 + \frac{135}{19}}{10} = \frac{99}{19}</math>, and the solution is <math>m + n = 118</math>.
(needs to be finished)
 
  
 
== See also ==
 
== See also ==
* [[1999_AIME_Problems/Problem_1|Previous Problem]]
+
{{AIME box|year=1999|num-b=1|num-a=3}}
* [[1999_AIME_Problems/Problem_3|Next Problem]]
+
 
* [[1999 AIME Problems]]
+
[[Category:Intermediate Geometry Problems]]

Revision as of 19:22, 8 March 2007

Problem

Consider the parallelogram with vertices $\displaystyle (10,45),$ $\displaystyle (10,114),$ $\displaystyle (28,153),$ and $\displaystyle (28,84).$ A line through the origin cuts this figure into two congruent polygons. The slope of the line is $\displaystyle m/n,$ where $\displaystyle m_{}$ and $\displaystyle n_{}$ are relatively prime positive integers. Find $\displaystyle m+n.$

Solution

Let the first point on the line $x=10$ be $(10,45+a)$ where a is the height above $(10,45)$. Let the second point on the line $x=28$ be $(28, 153-a)$. For two given points, the line will pass the origin iff the coordinates are proportional (such that $\frac{y_1}{x_1} = \frac{y_2}{x_2}$). Then, we can write that $\frac{45 + a}{10} = \frac{153 - a}{28}$. Solving for $a$ yields that $\displaystyle 1530 - 10a = 1260 + 28a$, so $a=\frac{270}{38}=\frac{135}{19}$. The slope of the line (since it passes through the origin) is $\frac{45 + \frac{135}{19}}{10} = \frac{99}{19}$, and the solution is $m + n = 118$.

See also

1999 AIME (ProblemsAnswer KeyResources)
Preceded by
Problem 1
Followed by
Problem 3
1 2 3 4 5 6 7 8 9 10 11 12 13 14 15
All AIME Problems and Solutions